Show that $left|frac{alpha - beta}{1-bar{alpha}beta}right| < 1$ when $|alpha|,|beta| < 1$












7












$begingroup$


This is the question I'm stumbling with:






When $|alpha| < 1$ and $|beta| < 1$, show that:

$$left|cfrac{alpha - beta}{1-bar{alpha}beta}right| < 1$$





The chapter that contains this question contains (among others) the triangle inequalities:



$$left||z_1| - |z_2|right| le |z_1 + z_2| le |z_1| + |z_2| $$



I've tried to use the triangle inequalities to increase the dividend and/or decrease the divisor:



$$left|cfrac{alpha - beta}{1-bar{alpha}beta}right| < cfrac{|alpha| +|beta|}{left|1-|bar{alpha}beta|right|}$$



But it's not clear if or why that would be smaller than one. I've also tried to multiply the equation by the conjugated divisor $cfrac{1-alphabar{beta}}{1-alphabar{beta}}$, which gives a real divisor, but the equation does not appear solvable.



Any hint would be much appreciated.










share|cite|improve this question











$endgroup$

















    7












    $begingroup$


    This is the question I'm stumbling with:






    When $|alpha| < 1$ and $|beta| < 1$, show that:

    $$left|cfrac{alpha - beta}{1-bar{alpha}beta}right| < 1$$





    The chapter that contains this question contains (among others) the triangle inequalities:



    $$left||z_1| - |z_2|right| le |z_1 + z_2| le |z_1| + |z_2| $$



    I've tried to use the triangle inequalities to increase the dividend and/or decrease the divisor:



    $$left|cfrac{alpha - beta}{1-bar{alpha}beta}right| < cfrac{|alpha| +|beta|}{left|1-|bar{alpha}beta|right|}$$



    But it's not clear if or why that would be smaller than one. I've also tried to multiply the equation by the conjugated divisor $cfrac{1-alphabar{beta}}{1-alphabar{beta}}$, which gives a real divisor, but the equation does not appear solvable.



    Any hint would be much appreciated.










    share|cite|improve this question











    $endgroup$















      7












      7








      7


      2



      $begingroup$


      This is the question I'm stumbling with:






      When $|alpha| < 1$ and $|beta| < 1$, show that:

      $$left|cfrac{alpha - beta}{1-bar{alpha}beta}right| < 1$$





      The chapter that contains this question contains (among others) the triangle inequalities:



      $$left||z_1| - |z_2|right| le |z_1 + z_2| le |z_1| + |z_2| $$



      I've tried to use the triangle inequalities to increase the dividend and/or decrease the divisor:



      $$left|cfrac{alpha - beta}{1-bar{alpha}beta}right| < cfrac{|alpha| +|beta|}{left|1-|bar{alpha}beta|right|}$$



      But it's not clear if or why that would be smaller than one. I've also tried to multiply the equation by the conjugated divisor $cfrac{1-alphabar{beta}}{1-alphabar{beta}}$, which gives a real divisor, but the equation does not appear solvable.



      Any hint would be much appreciated.










      share|cite|improve this question











      $endgroup$




      This is the question I'm stumbling with:






      When $|alpha| < 1$ and $|beta| < 1$, show that:

      $$left|cfrac{alpha - beta}{1-bar{alpha}beta}right| < 1$$





      The chapter that contains this question contains (among others) the triangle inequalities:



      $$left||z_1| - |z_2|right| le |z_1 + z_2| le |z_1| + |z_2| $$



      I've tried to use the triangle inequalities to increase the dividend and/or decrease the divisor:



      $$left|cfrac{alpha - beta}{1-bar{alpha}beta}right| < cfrac{|alpha| +|beta|}{left|1-|bar{alpha}beta|right|}$$



      But it's not clear if or why that would be smaller than one. I've also tried to multiply the equation by the conjugated divisor $cfrac{1-alphabar{beta}}{1-alphabar{beta}}$, which gives a real divisor, but the equation does not appear solvable.



      Any hint would be much appreciated.







      complex-analysis inequality complex-numbers






      share|cite|improve this question















      share|cite|improve this question













      share|cite|improve this question




      share|cite|improve this question








      edited Dec 10 '17 at 9:50









      Did

      247k23222457




      247k23222457










      asked Sep 26 '13 at 16:50









      AndomarAndomar

      286210




      286210






















          1 Answer
          1






          active

          oldest

          votes


















          6












          $begingroup$

          Hint:
          $$
          left| frac{alpha-beta}{1-baralphabeta}right| < 1 Leftrightarrow |alpha-beta|^2 < |1-baralphabeta|^2.
          $$



          Expand both sides, remembering that $|z|^2 = zbar z$ and simplify. That should get you where you want to be after some algebra.






          share|cite|improve this answer









          $endgroup$













          • $begingroup$
            Interesting, this hint gets me to $|alpha|^2 + |beta|^2 < 1 + |alpha|^2 |beta|^2$. Any hint from there?
            $endgroup$
            – Andomar
            Sep 26 '13 at 18:07










          • $begingroup$
            @Andomar What is $(1-|alpha|^2)(1-|beta|^2)$?
            $endgroup$
            – mrf
            Sep 26 '13 at 18:33












          • $begingroup$
            Very helpful, I like the partial answers! Thanks.
            $endgroup$
            – Andomar
            Sep 26 '13 at 18:44











          Your Answer





          StackExchange.ifUsing("editor", function () {
          return StackExchange.using("mathjaxEditing", function () {
          StackExchange.MarkdownEditor.creationCallbacks.add(function (editor, postfix) {
          StackExchange.mathjaxEditing.prepareWmdForMathJax(editor, postfix, [["$", "$"], ["\\(","\\)"]]);
          });
          });
          }, "mathjax-editing");

          StackExchange.ready(function() {
          var channelOptions = {
          tags: "".split(" "),
          id: "69"
          };
          initTagRenderer("".split(" "), "".split(" "), channelOptions);

          StackExchange.using("externalEditor", function() {
          // Have to fire editor after snippets, if snippets enabled
          if (StackExchange.settings.snippets.snippetsEnabled) {
          StackExchange.using("snippets", function() {
          createEditor();
          });
          }
          else {
          createEditor();
          }
          });

          function createEditor() {
          StackExchange.prepareEditor({
          heartbeatType: 'answer',
          autoActivateHeartbeat: false,
          convertImagesToLinks: true,
          noModals: true,
          showLowRepImageUploadWarning: true,
          reputationToPostImages: 10,
          bindNavPrevention: true,
          postfix: "",
          imageUploader: {
          brandingHtml: "Powered by u003ca class="icon-imgur-white" href="https://imgur.com/"u003eu003c/au003e",
          contentPolicyHtml: "User contributions licensed under u003ca href="https://creativecommons.org/licenses/by-sa/3.0/"u003ecc by-sa 3.0 with attribution requiredu003c/au003e u003ca href="https://stackoverflow.com/legal/content-policy"u003e(content policy)u003c/au003e",
          allowUrls: true
          },
          noCode: true, onDemand: true,
          discardSelector: ".discard-answer"
          ,immediatelyShowMarkdownHelp:true
          });


          }
          });














          draft saved

          draft discarded


















          StackExchange.ready(
          function () {
          StackExchange.openid.initPostLogin('.new-post-login', 'https%3a%2f%2fmath.stackexchange.com%2fquestions%2f506058%2fshow-that-left-frac-alpha-beta1-bar-alpha-beta-right-1-when%23new-answer', 'question_page');
          }
          );

          Post as a guest















          Required, but never shown

























          1 Answer
          1






          active

          oldest

          votes








          1 Answer
          1






          active

          oldest

          votes









          active

          oldest

          votes






          active

          oldest

          votes









          6












          $begingroup$

          Hint:
          $$
          left| frac{alpha-beta}{1-baralphabeta}right| < 1 Leftrightarrow |alpha-beta|^2 < |1-baralphabeta|^2.
          $$



          Expand both sides, remembering that $|z|^2 = zbar z$ and simplify. That should get you where you want to be after some algebra.






          share|cite|improve this answer









          $endgroup$













          • $begingroup$
            Interesting, this hint gets me to $|alpha|^2 + |beta|^2 < 1 + |alpha|^2 |beta|^2$. Any hint from there?
            $endgroup$
            – Andomar
            Sep 26 '13 at 18:07










          • $begingroup$
            @Andomar What is $(1-|alpha|^2)(1-|beta|^2)$?
            $endgroup$
            – mrf
            Sep 26 '13 at 18:33












          • $begingroup$
            Very helpful, I like the partial answers! Thanks.
            $endgroup$
            – Andomar
            Sep 26 '13 at 18:44
















          6












          $begingroup$

          Hint:
          $$
          left| frac{alpha-beta}{1-baralphabeta}right| < 1 Leftrightarrow |alpha-beta|^2 < |1-baralphabeta|^2.
          $$



          Expand both sides, remembering that $|z|^2 = zbar z$ and simplify. That should get you where you want to be after some algebra.






          share|cite|improve this answer









          $endgroup$













          • $begingroup$
            Interesting, this hint gets me to $|alpha|^2 + |beta|^2 < 1 + |alpha|^2 |beta|^2$. Any hint from there?
            $endgroup$
            – Andomar
            Sep 26 '13 at 18:07










          • $begingroup$
            @Andomar What is $(1-|alpha|^2)(1-|beta|^2)$?
            $endgroup$
            – mrf
            Sep 26 '13 at 18:33












          • $begingroup$
            Very helpful, I like the partial answers! Thanks.
            $endgroup$
            – Andomar
            Sep 26 '13 at 18:44














          6












          6








          6





          $begingroup$

          Hint:
          $$
          left| frac{alpha-beta}{1-baralphabeta}right| < 1 Leftrightarrow |alpha-beta|^2 < |1-baralphabeta|^2.
          $$



          Expand both sides, remembering that $|z|^2 = zbar z$ and simplify. That should get you where you want to be after some algebra.






          share|cite|improve this answer









          $endgroup$



          Hint:
          $$
          left| frac{alpha-beta}{1-baralphabeta}right| < 1 Leftrightarrow |alpha-beta|^2 < |1-baralphabeta|^2.
          $$



          Expand both sides, remembering that $|z|^2 = zbar z$ and simplify. That should get you where you want to be after some algebra.







          share|cite|improve this answer












          share|cite|improve this answer



          share|cite|improve this answer










          answered Sep 26 '13 at 16:57









          mrfmrf

          37.4k54685




          37.4k54685












          • $begingroup$
            Interesting, this hint gets me to $|alpha|^2 + |beta|^2 < 1 + |alpha|^2 |beta|^2$. Any hint from there?
            $endgroup$
            – Andomar
            Sep 26 '13 at 18:07










          • $begingroup$
            @Andomar What is $(1-|alpha|^2)(1-|beta|^2)$?
            $endgroup$
            – mrf
            Sep 26 '13 at 18:33












          • $begingroup$
            Very helpful, I like the partial answers! Thanks.
            $endgroup$
            – Andomar
            Sep 26 '13 at 18:44


















          • $begingroup$
            Interesting, this hint gets me to $|alpha|^2 + |beta|^2 < 1 + |alpha|^2 |beta|^2$. Any hint from there?
            $endgroup$
            – Andomar
            Sep 26 '13 at 18:07










          • $begingroup$
            @Andomar What is $(1-|alpha|^2)(1-|beta|^2)$?
            $endgroup$
            – mrf
            Sep 26 '13 at 18:33












          • $begingroup$
            Very helpful, I like the partial answers! Thanks.
            $endgroup$
            – Andomar
            Sep 26 '13 at 18:44
















          $begingroup$
          Interesting, this hint gets me to $|alpha|^2 + |beta|^2 < 1 + |alpha|^2 |beta|^2$. Any hint from there?
          $endgroup$
          – Andomar
          Sep 26 '13 at 18:07




          $begingroup$
          Interesting, this hint gets me to $|alpha|^2 + |beta|^2 < 1 + |alpha|^2 |beta|^2$. Any hint from there?
          $endgroup$
          – Andomar
          Sep 26 '13 at 18:07












          $begingroup$
          @Andomar What is $(1-|alpha|^2)(1-|beta|^2)$?
          $endgroup$
          – mrf
          Sep 26 '13 at 18:33






          $begingroup$
          @Andomar What is $(1-|alpha|^2)(1-|beta|^2)$?
          $endgroup$
          – mrf
          Sep 26 '13 at 18:33














          $begingroup$
          Very helpful, I like the partial answers! Thanks.
          $endgroup$
          – Andomar
          Sep 26 '13 at 18:44




          $begingroup$
          Very helpful, I like the partial answers! Thanks.
          $endgroup$
          – Andomar
          Sep 26 '13 at 18:44


















          draft saved

          draft discarded




















































          Thanks for contributing an answer to Mathematics Stack Exchange!


          • Please be sure to answer the question. Provide details and share your research!

          But avoid



          • Asking for help, clarification, or responding to other answers.

          • Making statements based on opinion; back them up with references or personal experience.


          Use MathJax to format equations. MathJax reference.


          To learn more, see our tips on writing great answers.




          draft saved


          draft discarded














          StackExchange.ready(
          function () {
          StackExchange.openid.initPostLogin('.new-post-login', 'https%3a%2f%2fmath.stackexchange.com%2fquestions%2f506058%2fshow-that-left-frac-alpha-beta1-bar-alpha-beta-right-1-when%23new-answer', 'question_page');
          }
          );

          Post as a guest















          Required, but never shown





















































          Required, but never shown














          Required, but never shown












          Required, but never shown







          Required, but never shown

































          Required, but never shown














          Required, but never shown












          Required, but never shown







          Required, but never shown







          Popular posts from this blog

          Mario Kart Wii

          What does “Dominus providebit” mean?

          Antonio Litta Visconti Arese